A geometric sequence has Determine a and r so that the sequence has the formula an = a · a = Number r = Number a3 = 200, a4 = 2,000, a.pn-1. a5 = 20,000,.

Answers

Answer 1

For a geometric sequence given three terms: a3 = 200, a4 = 2,000, and a5 = 20,000. We need to determine the common ratio, r, and the first term, a, so that the sequence follows the formula an = a * rn-1.

To find the values of a and r, we can use the given terms of the  sequence. Let's start with the equation for the fourth term, a4 = a * r^3 = 2,000. Similarly, we have a5 = a * r^4 = 20,000.

Dividing these two equations, we get (a5 / a4) = (a * r^4) / (a * r^3) = r. Therefore, we know that r = (a5 / a4). Now, let's substitute the value of r into the equation for the third term, a3 = a * r^2 = 200. We can rewrite this equation as a = (a3 / r^2).

Finally, we have found the values of a and r for the geometric sequence. a = (a3 / r^2) and r = (a5 / a4). Substituting the given values, we can calculate the specific values of a and r.

To learn more about geometric sequence click here : brainly.com/question/11266123

#SPJ11


Related Questions

The value of C that satisfy mean value theorem for f(x)=x²³ −x on the interval [0, 2] is: a) {1} a) B3} ©

Answers

The value of C that satisfies the mean value theorem for f(x) = x²³ − x on the interval [0, 2] is 1.174. So the option is none of the above.

The mean value theorem states that if a function f is continuous on the closed interval [a, b] and differentiable on the open interval (a, b), then there is at least one point c in (a, b) such that

f′(c)=(f(b)−f(a))/(b−a).

The given function is

f(x)=x²³ −x.

The function is continuous on the interval [0, 2] and differentiable on the open interval (0, 2).

Therefore, by mean value theorem, we know that there exists a point c in (0, 2) such that

f′(c)=(f(2)−f(0))/(2−0).

We need to find the value of C satisfying the theorem.

So we will start by calculating the derivative of f(x).

f′(x)=23x²² −1

According to the theorem, we can write:

23c²² −1 = (2²³ − 0²³ )/(2 − 0)

23c²² − 1 = 223

23c²² = 224

[tex]c = (224)^(1/22)[/tex]

c ≈ 1.174

Therefore, the value of C that satisfies the mean value theorem for f(x) = x²³ − x on the interval [0, 2] is approximately 1.174, which is not one of the answer choices provided.

Know more about the mean value theorem

https://brainly.com/question/30403137

#SPJ11

Find the volume of the solid of intersection of the two right circular cylinders of radius r whose axes meet at right angles.

Answers

The solid of intersection of the two right circular cylinders of radius r whose axes meet at right angles is known as a Steiner's Reversed Cycloid. It has a volume of V=16πr³/9. The intersection volume between two identical cylinders whose axes meet at right angles is called a Steiner solid (sometimes also referred to as a Steinmetz solid).

To find the volume of a Steiner solid, you must first define the radii of the two cylinders. The radii of the cylinders in this question are r. You must now compute the volume of the solid formed by the intersection of the two cylinders, which is the Steiner solid.

A method for determining the volume of the Steiner solid formed by the intersection of two cylinders whose axes meet at right angles is shown below. You can use any unit of measure, but be sure to use the same unit of measure for each length measurement. V=16πr³/9 is the formula for finding the volume of the Steiner solid for two right circular cylinders of the same radius r and whose axes meet at right angles. You can do this by subtracting the volumes of the two half-cylinders that are formed when the two cylinders intersect. The height of each of these half-cylinders is equal to the diameter of the circle from which the cylinder was formed, which is 2r. Each of these half-cylinders is then sliced in half to produce two quarter-cylinders. These quarter-cylinders are then used to construct a sphere of radius r, which is then divided into 9 equal volume pyramids, three of which are removed to create the Steiner solid.

Volume of half-cylinder: V1 = 1/2πr² * 2r

= πr³

Volume of quarter-cylinder: V2 = 1/4πr² * 2r

= πr³/2

Volume of sphere: V3 = 4/3πr³

Volume of one-eighth of the sphere: V4 = 1/8 * 4/3πr³

= 1/6πr³

Volume of the Steiner solid = 4V4 - 3V2

= (4/6 - 3/2)πr³

= 16/6 - 9/6

= 7/3πr³

= 2.333πr³ ≈ 7.33r³ (in terms of r³)

To know more about right angles visit :

https://brainly.com/question/3770177

#SPJ11

Find the average value of f(x) = xsec²(x²) on the interval | 0, [4] 2

Answers

The average value of f(x) = xsec²(x²) on the interval [0,2] is approximately 0.418619.

The average value of a function f(x) on an interval [a, b] is given by the formula:

f_avg = (1/(b-a)) * ∫[a,b] f(x) dx

In this case, we want to find the average value of f(x) = xsec²(x²) on the interval [0,2]. So we can compute it as:

f_avg = (1/(2-0)) * ∫[0,2] xsec²(x²) dx

To solve the integral, we can make a substitution. Let u = x², then du/dx = 2x, and dx = du/(2x). Substituting these expressions in the integral, we have:

f_avg = (1/2) * ∫[0,2] (1/(2x))sec²(u) du

Simplifying further, we have:

f_avg = (1/4) * ∫[0,2] sec²(u)/u du

Using the formula for the integral of sec²(u) from the table of integrals, we have:

f_avg = (1/4) * [(tan(u) * ln|tan(u)+sec(u)|) + C] |_0^4

Evaluating the integral and applying the limits, we get:

f_avg = (1/4) * [(tan(4) * ln|tan(4)+sec(4)|) - (tan(0) * ln|tan(0)+sec(0)|)]

Calculating the numerical values, we find:

f_avg ≈ (0.28945532058739433 * 1.4464994978877052) ≈ 0.418619

Therefore, the average value of f(x) = xsec²(x²) on the interval [0,2] is approximately 0.418619.

Learn more about average value

https://brainly.com/question/13391650

#SPJ11

A car is travelling with varying speed, and at the moment t = 0 the speed is 100 km/h. The car gradually slows down according to the formula L(t) = at bt², t≥0, - where L(t) is the distance travelled along the road and b = 90 km/h². The value of a is not given, but you can find it. Using derivative, find the time moment when the car speed becomes 10 km/h. Find the acceleration of the car at that moment.

Answers

The acceleration of the car at that moment is -45 km/h².

Given function:

L(t) = at + bt² at time

t = 0,

L(0) = 0 (initial position of the car)

Now, differentiating L(t) w.r.t t, we get:

v(t) = L'(t) = a + 2bt

Also, given that,

v(0) = 100 km/h

Substituting t = 0,

we get: v(0) = a = 100 km/h

Also, it is given that v(t) = 10 km/h at some time t.

Therefore, we can write:

v(t) = a + 2bt = 10 km/h

Substituting the value of a,

we get:

10 km/h = 100 km/h + 2bt2

bt = -90 km/h

b = -45 km/h²

As b is negative, the car is decelerating.

Now, substituting the value of b in the expression for v(t),

we get: v(t) = 100 - 45t km/h At t = ? (the moment when the speed of the car becomes 10 km/h),

we have: v(?) = 10 km/h100 - 45t = 10 km/h

t = 1.8 h

The time moment when the car speed becomes 10 km/h is 1.8 h.

The acceleration of the car at that moment can be found by differentiating the expression for

v(t):a(t) = v'(t) = d/dt (100 - 45t) = -45 km/h²

Therefore, the acceleration of the car at that moment is -45 km/h².

To know more about acceleration visit:

https://brainly.com/question/2303856

#SPJ11

Given the following set of ordered pairs: [4] f={(-2,3), (-1, 1), (0, 0), (1,-1), (2,-3)} g = {(-3,1),(-1,-2), (0, 2), (2, 2), (3, 1)) a) State (f+g)(x) b) State (f+g)(x) c) Find (fog)(3) d) Find (gof)(-2)

Answers

To find (f+g)(x), we need to add the corresponding y-values of f and g for each x-value.

a) (f+g)(x) = {(-2, 3) + (-3, 1), (-1, 1) + (-1, -2), (0, 0) + (0, 2), (1, -1) + (2, 2), (2, -3) + (3, 1)}

Expanding each pair of ordered pairs:

(f+g)(x) = {(-5, 4), (-2, -1), (0, 2), (3, 1), (5, -2)}

b) To state (f-g)(x), we need to subtract the corresponding y-values of f and g for each x-value.

(f-g)(x) = {(-2, 3) - (-3, 1), (-1, 1) - (-1, -2), (0, 0) - (0, 2), (1, -1) - (2, 2), (2, -3) - (3, 1)}

Expanding each pair of ordered pairs:

(f-g)(x) = {(1, 2), (0, 3), (0, -2), (-1, -3), (-1, -4)}

c) To find (f∘g)(3), we need to substitute x=3 into g first, and then use the result as the input for f.

(g(3)) = (2, 2)Substituting (2, 2) into f:

(f∘g)(3) = f(2, 2)

Checking the given set of ordered pairs in f, we find that (2, 2) is not in f. Therefore, (f∘g)(3) is undefined.

d) To find (g∘f)(-2), we need to substitute x=-2 into f first, and then use the result as the input for g.

(f(-2)) = (-3, 1)Substituting (-3, 1) into g:

(g∘f)(-2) = g(-3, 1)

Checking the given set of ordered pairs in g, we find that (-3, 1) is not in g. Therefore, (g∘f)(-2) is undefined.

Learn more about function  here:

brainly.com/question/11624077

#SPJ11

Solve the equation by extracting the square roots. List both the exact solution and its approximation round x² = 49 X = (smaller value) X = (larger value) Need Help? 10. [0/0.26 Points] DETAILS PREVIOUS ANSWERS LARCOLALG10 1.4.021. Solve the equation by extracting the square roots. List both the exact solution and its approximation rounded +² = 19 X = X (smaller value) X = X (larger value) Need Help? Read It Read It nd its approximation X = X = Need Help? 12. [-/0.26 Points] DETAILS LARCOLALG10 1.4.026. Solve the equation by extracting the square roots. List both the exact solution and its approximation rour (x - 5)² = 25 X = (smaller value) X = (larger value) x² = 48 Need Help? n Read It Read It (smaller value) (larger value) Watch It Watch It

Answers

The exact solution is x = ±√48, but if you need an approximation, you can use a calculator to find the decimal value. x ≈ ±6.928

1. x² = 49

The square root of x² = √49x = ±7

Therefore, the smaller value is -7, and the larger value is 7.2. (x - 5)² = 25

To solve this equation by extracting square roots, you need to isolate the term that is being squared on one side of the equation.

x - 5 = ±√25x - 5

= ±5x = 5 ± 5

x = 10 or

x = 0

We have two possible solutions, x = 10 and x = 0.3. x² = 48

The square root of x² = √48

The number inside the square root is not a perfect square, so we can't simplify the expression.

The exact solution is x = ±√48, but if you need an approximation, you can use a calculator to find the decimal value.

x ≈ ±6.928

To know more about square root visit:

https://brainly.com/question/29286039

#SPJ11

Find solutions for your homework
Find solutions for your homework
mathalgebraalgebra questions and answers
This problem has been solved!
You'll get a detailed solution from a subject matter expert that helps you learn core concepts.
See Answer
Question:
Evaluating and Solving an Equation Application
Identify the information given to you in the application problem below. Use that information to answer the questions that follow.
Round your answers to two decimal places as needed.
The cost to fill your motor home's propane tank is determined by the function C
(
g
)
=
3.49
g
where C
(
g
)
is the output (cost in $) and g
is the input (gallons of gas). The propane tank can hold a maximum of 21 gallons
Calculate C
(
4
)
: C
(
4
)
=
Write your answer as an Ordered Pair:
Complete the following sentence to explain the meaning of #1 and #2:
The cost to purchase gallons of propane is dollars

Answers

In this case, the function C(g) calculates the cost (output) based on the number of gallons (input).  Therefore, the cost to fill the motor home's propane tank with 4 gallons of gas is $13.96.

To evaluate C(4), we substitute the value of 4 into the function C(g). By doing so, we obtain C(4) = 3.49 * 4 = 13.96. Therefore, the cost to fill the motor home's propane tank with 4 gallons of gas is $13.96.

Regarding the meaning of #1 and #2, #1 refers to the input value or the number of gallons of propane being purchased, while #2 represents the output value or the cost of purchasing those gallons of propane in dollars. In this case, the function C(g) calculates the cost (output) based on the number of gallons (input).

So, when we say "The cost to purchase gallons of propane is dollars," it means that the function C(g) gives us the cost in dollars based on the number of gallons of propane being purchased.

Learn more about function here:

https://brainly.com/question/30721594

#SPJ11

If the rational function y = r(x) has the vertical asymptote x = 7, then as x --> 7^+, either y --> ____________

Answers

If the rational function y = r(x) has the vertical asymptote x = 7, then as x → 7+ (approaches 7 from the right-hand side), either y → ∞ (approaches infinity).

The behavior of a function, f(x), around vertical asymptotes is essential to understand the graph of rational functions, especially when we need to sketch them by hand.

The vertical asymptote at x = a is the line where f(x) → ±∞ as x → a. The limit as x approaches a from the right is f(x) → +∞, and from the left, f(x) → -∞.

For example, if the rational function has a vertical asymptote at x = 7,

The limit as x approaches 7 from the right is y → ∞ (approaches infinity). That is, as x gets closer and closer to 7 from the right, the value of y gets larger and larger.

Thus, as x → 7+ , either y → ∞ (approaches infinity).

Learn more about asymptote at

https://brainly.com/question/23412972?

#SPJ11

Two Points A (-2, -1) and B (8, 5) are given. If C is a point on the y-axis such that AC-BC, then the coordinates of C is: A. (3,2) B. (0, 2) C. (0,7) D. (4,2) 2. Given two points A (0, 4) and B (3, 7), what is the angle of inclination that the line segment A makes with the positive x-axis? A. 90⁰ B. 60° C. 45° D. 30°

Answers

The coordinates of C are (0, 2), and the angle of inclination that line AB makes with the positive x-axis is 45°.

1) Given two points A (-2, -1) and B (8, 5) on the plane. If C is a point on the y-axis such that AC-BC, then the coordinates of C is (0, 2). Given two points A (-2, -1) and B (8, 5) on the plane.

To find a point C on the y-axis such that AC-BC. So, we can say that C lies on the line passing through A and B, whose equation can be given by

y+1=(5+1)/(8+2)(x+2)y+1

y =3/2(x+2)

The point C lies on the y-axis. So, the x-coordinate of C will be 0. Substitute x=0 in the equation of the line passing through A and B to get

y+1=3/2(0+2)

y+1=3y/2

The coordinates of C are (0, 2).

Hence, the correct option is B. (0, 2).

2) Given two points, A (0, 4) and B (3, 7). The angle of inclination that line segment A makes with the positive x-axis is 45°. The inclination of a line is the angle between the positive x-axis and the line. A line with inclination makes an angle of 90° − with the negative x-axis.

Therefore, the angle of inclination that line AB makes with the positive x-axis is given by

tan = (y2 − y1) / (x2 − x1)

tan = (7 − 4) / (3 − 0)

tan = 3/3 = 1

Therefore, = tan⁻¹(1) = 45°

Hence, the correct option is C. 45°

The coordinates of C are (0, 2), and the angle of inclination that line AB makes with the positive x-axis is 45°.

To know more about the angle of inclination, visit:

brainly.com/question/31329503

#SPJ11

The following limit represents the slope of a curve y=f(x) at the point (a,f(a)). Determine a function f and a number a; then, calculate the limit. √29+h-√29 lim h-0 h GA. Pix) Evh+x OB. f(x)=√h+x-√29 c. f(x)=√x *D. f(x)=√29 Determine the number a. a= (Type an exact answer, using radicals as needed.)

Answers

Answer:

From the limit expression √29+h-√29 lim h-0 h, we can simplify the numerator as:

√(29+h) - √29 = (√(29+h) - √29)(√(29+h) + √29)/(√(29+h) + √29)

= (29+h - 29)/(√(29+h) + √29)

= h/(√(29+h) + √29)

Thus the limit expression becomes:

lim h->0 h/(√(29+h) + √29)

To simplify this expression further, we can multiply the numerator and denominator by the conjugate of the denominator, which is (√(29+h) - √29):

lim h->0 h/(√(29+h) + √29) * (√(29+h) - √29)/(√(29+h) - √29)

= lim h->0 h(√(29+h) - √29)/((29+h) - 29)

= lim h->0 (√(29+h) - √29)/h

This is now in the form of a derivative, specifically the derivative of f(x) = √x evaluated at x = 29. Therefore, we can take f(x) = √x and a = 29, and the limit is the slope of the tangent line to the curve y = √x at x = 29.

To determine the value of the limit, we can use the definition of the derivative:

f'(29) = lim h->0 (f(29+h) - f(29))/h = lim h->0 (√(29+h) - √29)/h

This is the same limit expression we derived earlier. Therefore, f(x) = √x and a = 29, and the limit is f'(29) = lim h->0 (√(29+h) - √29)/h.

To calculate the limit, we can plug in h = 0 and simplify:

lim h->0 (√(29+h) - √29)/h

= lim h->0 ((√(29+h) - √29)/(h))(1/1)

= f'(29)

= 1/(2√29)

Thus, the function f(x) = √x and the number a = 29, and the limit is 1/(2√29).

The capacitor in an RC-circuit begins charging at t = 0. Its charge Q can be modelled as a function of time t by
Q(t) = a
where a and tc are constants with tc > 0. (We call tc the time constant.)
A) Determine the constant a if the capacitor eventually (as t → [infinity]) attains a charge of 2000 µF (microfarads).
B) If it takes 12 s to reach a 50% charge (i.e., 1000 µF), determine the time constant tc.
C) How long will it take for the capacitor to reach a 90% charge (i.e., 1800 µF)?

Answers

It will take approximately 2.303tc seconds for the capacitor to reach a 90% charge.

A) To determine the constant "a" for the capacitor to eventually attain a charge of 2000 µF (microfarads) as t approaches infinity, we set a equal to the capacitance value C, which is 2000 µF. Hence, the value of "a" is 2000 µF.

B) If it takes 12 s to reach a 50% charge (i.e., 1000 µF), we can determine the time constant "tc" using the formula Q(t) = a(1 − e^(-t/tc)).

When t equals tc, Q(tc) = a(1 − e^(-1)) = 0.63a.

We are given that Q(tc) = 0.5a. So, we have 0.5a = a(1 − e^(-1)).

Simplifying this equation, we find that tc = 12 s.

C) To find the time it takes for the capacitor to reach a 90% charge (i.e., 1800 µF), we need to solve for t in the equation Q(t) = 0.9a = 0.9 × 2000 = 1800 µF.

Using the formula Q(t) = a(1 − e^(-t/tc)), we have 0.9a = a(1 − e^(-t/tc)).

This simplifies to e^(-t/tc) = 0.1.

Taking the natural logarithm of both sides, we get -t/tc = ln(0.1).

Solving for t, we have t = tc ln(10) ≈ 2.303tc.

Thus, it will take approximately 2.303tc seconds for the capacitor to reach a 90% charge.

Learn more about capacitor

https://brainly.com/question/31627158

#SPJ11

Calculate the size of one of the interior angles of a regular heptagon (i.e. a regular 7-sided polygon) Enter the number of degrees to the nearest whole number in the box below. (Your answer should be a whole number, without a degrees sign.) Answer: Next page > < Previous page

Answers

The answer should be a whole number, without a degree sign and it is 129.

A regular polygon is a 2-dimensional shape whose angles and sides are congruent. The polygons which have equal angles and sides are called regular polygons. Here, the given polygon is a regular heptagon which has seven sides and seven equal interior angles. In order to calculate the size of one of the interior angles of a regular heptagon, we need to use the formula:

Interior angle of a regular polygon = (n - 2) x 180 / nwhere n is the number of sides of the polygon. For a regular heptagon, n = 7. Hence,Interior angle of a regular heptagon = (7 - 2) x 180 / 7= 5 x 180 / 7= 900 / 7

degrees= 128.57 degrees (rounded to the nearest whole number)

Therefore, the size of one of the interior angles of a regular heptagon is 129 degrees (rounded to the nearest whole number). Hence, the answer should be a whole number, without a degree sign and it is 129.

To know more about whole number visit:

https://brainly.com/question/29766862

#SPJ11

The tale to right gives the projections of the population of a country from 2000 to 2100. Answer parts (a) through (e) Year Population Year (millions) 2784 2000 2060 2010 3001 2070 2000 3205 2010 2900 3005 2000 240 3866 20 404 4 (a) Find a Iraar function that models a data, with equal to the number of years after 2000 d x) aquel to the population is mons *** (Use integers or decimals for any numbers in the expression Round to three decimal places as needed) () Find (76) 4701- Round to one decimal place as needed) State what does the value of 170) men OA The will be the projected population in year 2070, OB. The will be the projected population in year 2170 (e) What does this model predict the population to be in 20007 The population in year 2000 will be mikon (Round to one decimal place as needed.) How does this compare with the value for 2080 in the table? OA The value is not very close to the table value OB This value is tainly close to the table value. Put data set Population inition) 438.8 3146 906 1 6303 6742 Time Remaining 01:2018 Next Year The table to right gives the projections of the population of a country from 2000 to 2100 Arawer pants (a) through (e) Population Year (millions) 2060 2000 2784 2016 3001 2070 2000 3295 2060 2030 2000 2040 3804 2100 2060 4044 GO (a) Find a inear function that models this dats, with x equal to the number of years after 2000 and Ex equal to the population in milions *** (Use egers or decimals for any numbers in the expression. Round to three decimal places as needed) (b) Find (70) 470)(Round to one decimal place as needed) State what does the value of 70) mean OA. This will be the projected population in year 2010 OB. This will be the projected population in year 2170 (c) What does this model predict the population to be is 2007 million. The population in year 2080 will be (Round to one decimal place as needed) How does this compare with the value for 2080 in the table? OA This value is not very close to the table value OB This value is fairy close to the table value Ful dala Population ptions) 439 6 4646 506.1 530.3 575.2 Year 2000 2010 -2020 2030 2040 2050 Population Year (millions) 278.4 2060 308.1 2070 329.5 2080 360.5 2090 386.6 2100 404.4 . Full data set Population (millions) 439.8 464.6 506.1 536.3 575.2

Answers

The population projections for a country are given in a table. The linear function to model the data, determine the projected population in specific years, and compare the model's prediction with the values in the table.

To find a linear function that models the data, we can use the given population values and corresponding years. Let x represent the number of years after 2000, and let P(x) represent the population in millions. We can use the population values for 2000 and another year to determine the slope of the linear function.

Taking the population values for 2000 and 2060, we have two points (0, 2784) and (60, 3295). Using the slope-intercept form of a linear function, y = mx + b, where m is the slope and b is the y-intercept, we can calculate the slope as (3295 - 2784) / (60 - 0) = 8.517. Next, using the point (0, 2784) in the equation, we can solve for the y-intercept b = 2784. Therefore, the linear function that models the data is P(x) = 8.517x + 2784.

For part (b), we are asked to find P(70), which represents the projected population in the year 2070. Substituting x = 70 into the linear function, we get P(70) = 8.517(70) + 2784 = 3267.19 million. The value of P(70) represents the projected population in the year 2070.

In part (c), we need to determine the population prediction for the year 2007. Since the year 2007 is 7 years after 2000, we substitute x = 7 into the linear function to get P(7) = 8.517(7) + 2784 = 2805.819 million. The population prediction for the year 2007 is 2805.819 million.

For part (e), we compare the projected population for the year 2080 obtained from the linear function with the value in the table. Using x = 80 in the linear function, we find P(80) = 8.517(80) + 2784 = 3496.36 million. Comparing this with the table value for the year 2080, 329.5 million, we can see that the value obtained from the linear function (3496.36 million) is not very close to the table value (329.5 million).

Learn more about population here:

https://brainly.com/question/31598322

#SPJ11

Which of the following are the eigenvalues of (-12)² ? 0 1 ± 2i 0 1± √/2i O 2 + i O √2+i 4. (We will use the notation ☀ = dx/dt.) The solution of ï = kt with initial conditions (0) = 1 and (0) = -1 is given by kt3³ x(t)=1-t+ 6 x(t)=1-t+t² + kt³ x(t) = cost - sint + 6 x(t) = 2 cost - sint − 1 + kt³ 6 kt³ 6

Answers

The eigenvalues of (-12)² can be found by squaring the eigenvalues of -12.

The eigenvalues of -12 are the solutions to the equation λ = -12, where λ represents the eigenvalue.

Solving this equation, we have:

λ = -12.

Now, squaring both sides of the equation, we get:

λ² = (-12)² = 144.

Therefore, the eigenvalue of (-12)² is 144.

To summarize, the eigenvalue of (-12)² is 144.

Learn more about equation here:

https://brainly.com/question/29657983

#SPJ11

What are the last three digits of 1234^5678

Answers

The last three digits of 1234^5678 are 176.

Calculate the surface area generated by revolving the curve y=- 31/1 6366.4 O 2000 O 2026.5 O 2026.5 A -x³. , from x = 0 to x = 3 about the x-axis.

Answers

To calculate the surface area generated by revolving the curve y = -31/16366.4x³, from x = 0 to x = 3 about the x-axis, we can use the formula for surface area of a curve obtained through revolution. The resulting surface area will provide an indication of the extent covered by the curve when rotated.

In order to find the surface area generated by revolving the given curve about the x-axis, we can use the formula for surface area of a curve obtained through revolution, which is given by the integral of 2πy√(1 + (dy/dx)²) dx. In this case, the curve is y = -31/16366.4x³, and we need to evaluate the integral from x = 0 to x = 3.

First, we need to calculate the derivative of y with respect to x, which gives us dy/dx = -31/5455.467x². Plugging this value into the formula, we get the integral of 2π(-31/16366.4x³)√(1 + (-31/5455.467x²)²) dx from x = 0 to x = 3.

Evaluating this integral will give us the surface area generated by revolving the curve. By performing the necessary calculations, the resulting value will provide the desired surface area, indicating the extent covered by the curve when rotated around the x-axis.

Learn more about curve here : brainly.com/question/30511233

#SPJ11

h(x) = ln x+1) x - 1 f(x)=√x² - 1 sec-¹ X

Answers

The solution of H(x) = ln(x+1)/x - 1 and f(x) = √x² - 1 sec-¹ x is x = 1. The direct solution is found by first finding the intersection of the two functions. This can be done by setting the two functions equal to each other and solving for x.

The resulting equation is:

```

ln(x+1)/x - 1 = √x² - 1 sec-¹ x

```

This equation can be solved using the Lambert W function. The Lambert W function is a special function that solves equations of the form:

```

z = e^w

```

In this case, z = ln(x+1)/x - 1 and w = √x² - 1 sec-¹ x. The Lambert W function has two branches, W_0 and W_1. The W_0 branch is the principal branch and it is the branch that is used in this case. The solution for x is then given by:

```

x = -W_0(ln(x+1)/x - 1)

```

The Lambert W function is not an elementary function, so it cannot be solved exactly. However, it can be approximated using numerical methods. The approximation that is used in this case is:

```

x = 1 + 1/(1 + ln(x+1))

```

This approximation is accurate to within 10^-12 for all values of x. The resulting solution is x = 1.

Learn more about function here:

brainly.com/question/30721594

#SPJ11

Algebra The characteristic polynomial of the matrix 5 -2 A= -2 8 -2 4 -2 5 is X(X - 9)². The vector 1 is an eigenvector of A. -6 Find an orthogonal matrix P that diagonalizes A. and verify that PAP is diagonal

Answers

To diagonalize matrix A, we need to find an orthogonal matrix P. Given that the characteristic polynomial of A is X(X - 9)² and the vector [1 -6] is an eigenvector.

The given characteristic polynomial X(X - 9)² tells us that the eigenvalues of matrix A are 0, 9, and 9. We are also given that the vector [1 -6] is an eigenvector of A. To diagonalize A, we need to find two more eigenvectors corresponding to the eigenvalue 9.

Let's find the remaining eigenvectors:

For the eigenvalue 0, we solve the equation (A - 0I)v = 0, where I is the identity matrix and v is the eigenvector. Solving this equation, we find v₁ = [2 -1 1]ᵀ.

For the eigenvalue 9, we solve the equation (A - 9I)v = 0. Solving this equation, we find v₂ = [1 2 2]ᵀ and v₃ = [1 0 1]ᵀ.

Next, we normalize the eigenvectors to obtain the orthogonal matrix P:

P = [v₁/norm(v₁) v₂/norm(v₂) v₃/norm(v₃)]

  = [2√6/3 -√6/3 √6/3; √6/3 2√6/3 0; √6/3 2√6/3 √6/3]

Now, we can verify that PAP is diagonal:

PAPᵀ = [2√6/3 -√6/3 √6/3; √6/3 2√6/3 0; √6/3 2√6/3 √6/3]

      × [5 -2 8; -2 4 -2; 5 -2 5]

      × [2√6/3 √6/3 √6/3; -√6/3 2√6/3 2√6/3; √6/3 0 √6/3]

    = [0 0 0; 0 9 0; 0 0 9]

As we can see, PAPᵀ is a diagonal matrix, confirming that P diagonalizes matrix A.

Learn more about orthogonal here:

https://brainly.com/question/27749918

#SPJ11

The mess in a house can be measured by M (t). Assume that at M (0)=0, the house starts out clean. Over time the rate of change in the mess is proportional to 100-M. A completely messy house has a value of 100. What is the particular solution of M(t), if k is a constant? OM= 100-100 OM 100+100et OM 100-100e-t OM = 100+ 100e

Answers

The mess in a house can be modeled by the equation M(t) = 100 - 100e^(-kt), where k is a constant. This equation shows that the mess will increase over time, but at a decreasing rate. The house will never be completely messy, but it will approach 100 as t approaches infinity.

The initial condition M(0) = 0 tells us that the house starts out clean. The rate of change of the mess is proportional to 100-M, which means that the mess will increase when M is less than 100 and decrease when M is greater than 100. The constant k determines how quickly the mess changes. A larger value of k will cause the mess to increase more quickly.

The equation shows that the mess will never be completely messy. This is because the exponential term e^(-kt) will never be equal to 0. As t approaches infinity, the exponential term will approach 0, but it will never reach it. This means that the mess will approach 100, but it will never reach it.

Learn more about exponential term here:

brainly.com/question/30240961

#SPJ11

Find the exact length of the curve. Need Help? Read It DETAILS Find the exact length of the curve. e +9 Need Help? SCALCET8 10.2.041. x = 3 + 6t², y = 9 + 4t³, 0 ≤t≤4 Watch It PREVIOUS ANSWERS 7.

Answers

The exact length of the curve is 8√3 + 16√6 units long.

We are given the parametric equations x = 3 + 6t² and y = 9 + 4t³. To determine the length of the curve, we can use the formula:

L = ∫[a, b] √(dx/dt)² + (dy/dt)² dt,

where a = 0 and b = 4.

Differentiating x and y with respect to t gives dx/dt = 12t and dy/dt = 12t².

Therefore, dx/dt² = 12 and dy/dt² = 24t.

Substituting these values into the length formula, we have:

L = ∫[0,4] √(12 + 24t) dt.

We can simplify the equation further:

L = ∫[0,4] √12 dt + ∫[0,4] √(24t) dt.

Evaluating the integrals, we get:

L = 2√3t |[0,4] + 4√6t²/2 |[0,4].

Simplifying this expression, we find:

L = 2√3(4) + 4√6(4²/2) - 0.

Therefore, the exact length of the curve is 8√3 + 16√6 units long.

The final answer is 8√3 + 16√6.

Learn more about curve

https://brainly.com/question/20488542

#SPJ11

Find the derivative of the following function. 5 2 y = 3x + 2x +x - 5 y'=0 C

Answers

The derivative of the function `y = 3x + 2x + x - 5` is `6x - 5`. This can be found using the sum rule, the power rule, and the constant rule of differentiation.

The sum rule states that the derivative of a sum of two functions is the sum of the derivatives of the two functions. In this case, the function `y` is the sum of three functions: `3x`, `2x`, and `x`. The derivatives of these three functions are `3`, `2`, and `1`, respectively. Therefore, the derivative of `y` is `3 + 2 + 1 = 6`.

The power rule states that the derivative of `x^n` is `n * x^(n - 1)`. In this case, the function `y` contains the terms `3x`, `2x`, and `x`. The exponents of these terms are `1`, `1`, and `0`, respectively. Therefore, the derivatives of these three terms are `3`, `2`, and `0`, respectively.

The constant rule states that the derivative of a constant is zero. In this case, the function `y` contains the constant term `-5`. Therefore, the derivative of this term is `0`.

Combining the results of the sum rule, the power rule, and the constant rule, we get that the derivative of `y` is `6x - 5`.

Learn more about differentiation here:

brainly.com/question/13958985

#SPJ11

what is the perimeter of square abcd? units units 28 units 37 units

Answers

The perimeter of square ABCD is 28 units.

The perimeter of a square is the sum of all its sides. In this case, we need to find the perimeter of square ABCD.

The question provides two possible answers: 28 units and 37 units. However, we can only choose one correct answer. To determine the correct answer, let's think step by step.

A square has all four sides equal in length. Therefore, if we know the length of one side, we can find the perimeter.

If the perimeter of the square is 28 units, that would mean each side is 28/4 = 7 units long. However, if the perimeter is 37 units, that would mean each side is 37/4 = 9.25 units long.

Since a side length of 9.25 units is not a whole number, it is unlikely to be the correct answer. Hence, the perimeter of square ABCD is most likely 28 units.

In conclusion, the perimeter of square ABCD is 28 units.

Know more about perimeter here,

https://brainly.com/question/7486523

#SPJ11

i=1 For each of integers n ≥ 0, let P(n) be the statement ni 2²=n·2n+2 +2. (a) i. Write P(0). ii. Determine if P(0) is true. (b) Write P(k). (c) Write P(k+1). (d) Show by mathematical induction that P(n) is true.

Answers

The statement P(-3/2) is invalid since n must be an integer greater than or equal to zero. As a result, our mathematical induction is complete.

For each of integers n ≥ 0, let P(n) be the statement n × 2² = n × 2^(n+2) + 2.(a)

i. Writing P(0).When n = 0, we have:

P(0) is equivalent to 0 × 2² = 0 × 2^(0+2) + 2.

This reduces to: 0 = 2, which is not true.

ii. Determining whether P(0) is true.

The answer is no.

(b) Writing P(k). For some k ≥ 0, we have:

P(k): k × 2²

= k × 2^(k+2) + 2.

(c) Writing P(k+1).

Now, we have:

P(k+1): (k+1) × 2²

= (k+1) × 2^(k+1+2) + 2.

(d) Show by mathematical induction that P(n) is true. By mathematical induction, we must now demonstrate that P(n) is accurate for all n ≥ 0.

We have previously discovered that P(0) is incorrect. As a result, we begin our mathematical induction with n = 1. Since n = 1, we have:

P(1): 1 × 2² = 1 × 2^(1+2) + 2.This becomes 4 = 4 + 2, which is valid.

Inductive step:

Assume that P(n) is accurate for some n ≥ 1 (for an arbitrary but fixed value). In this way, we want to demonstrate that P(n+1) is also true. Now we must demonstrate:

P(n+1): (n+1) × 2² = (n+1) × 2^(n+3) + 2.

We will begin with the left-hand side (LHS) to show that this is true.

LHS = (n+1) × 2² [since we are considering P(n+1)]LHS = (n+1) × 4 [since 2² = 4]

LHS = 4n+4

We will now begin on the right-hand side (RHS).

RHS = (n+1) × 2^(n+3) + 2 [since we are considering P(n+1)]

RHS = (n+1) × 8 + 2 [since 2^(n+3) = 8]

RHS = 8n+10

The equation LHS = RHS is what we want to accomplish.

LHS = RHS implies that:

4n+4 = 8n+10

Subtracting 4n from both sides, we obtain:

4 = 4n+10

Subtracting 10 from both sides, we get:

-6 = 4n

Dividing both sides by 4, we find

-3/2 = n.

The statement P(-3/2) is invalid since n must be an integer greater than or equal to zero. As a result, our mathematical induction is complete. The mathematical induction proof is complete, demonstrating that P(n) is accurate for all n ≥ 0.

To know more about mathematical induction, visit:

brainly.com/question/29503103

#SPJ11

Which is a better price: 5 for $1. 00, 4 for 85 cents, 2 for 25 cents, or 6 for $1. 10

Answers

Answer:

2 for 25 cents is a better price

what is the value of x​

plssss guys can somone help me

Answers

a. The value of x in the circle is 67 degrees.

b. The value of x in the circle is 24.

How to solve circle theorem?

If two chords intersect inside a circle, then the measure of the angle formed is one half the sum of the measure of the arcs intercepted by the angle and its vertical angle.

Therefore, using the chord intersection theorem,

a.

51 = 1 / 2 (x + 35)

51 = 1 / 2x + 35 / 2

51 - 35 / 2 = 0.5x

0.5x = 51 - 17.5

x = 33.5 / 0.5

x = 67 degrees

Therefore,

b.

If a tangent and a chord intersect at a point on a circle, then the measure of each angle formed is one-half the measure of its intercepted arc.

61 = 1 / 2 (10x + 1 - 5x + 1)

61 = 1 / 2 (5x + 2)

61 = 5 / 2 x + 1

60 = 5 / 2 x

cross multiply

5x = 120

x = 120 / 5

x = 24

learn more on circle theorem here: https://brainly.com/question/23769502

#SPJ1

Find the general solution of the differential equation x³ p+2x²y"+xy'-y = 0 X

Answers

The given differential equation is x³y" + 2x²y' + xy' - y = 0. We need to find the general solution for this differential equation.

To find the general solution, we can use the method of power series or assume a solution of the form y = ∑(n=0 to ∞) anxn, where an are coefficients to be determined.

First, we find the derivatives of y with respect to x:

y' = ∑(n=1 to ∞) nanxn-1,

y" = ∑(n=2 to ∞) n(n-1)anxn-2.

Substituting these derivatives into the differential equation, we have:

x³(∑(n=2 to ∞) n(n-1)anxn-2) + 2x²(∑(n=1 to ∞) nanxn-1) + x(∑(n=0 to ∞) nanxn) - (∑(n=0 to ∞) anxn) = 0.

Simplifying and re-arranging terms, we get:

∑(n=2 to ∞) n(n-1)anxn + 2∑(n=1 to ∞) nanxn + ∑(n=0 to ∞) nanxn - ∑(n=0 to ∞) anxn = 0.

Now, we equate the coefficients of like powers of x to obtain a recursion relation for the coefficients an.

For n = 0: -a₀ = 0, which gives a₀ = 0.

For n = 1: 2a₁ - a₁ = 0, which gives a₁ = 0.

For n ≥ 2: n(n-1)an + 2nan + nan - an = 0, which simplifies to: (n² + 2n + 1 - 1)an = 0.

Solving the above equation, we have: an = 0 for n ≥ 2.

Therefore, the general solution of the given differential equation is:

y(x) = a₀ + a₁x.

To learn more about derivatives  Click Here: brainly.com/question/25324584

#SPJ11

10) Determine whether the events of rolling a fair die two times are disjoint, independent, both, or neither. A) Disjoint. B) Exclusive. C) Independent. D) All of these. E) None of these.

Answers

The answer is option (C), that is, the events of rolling a fair die two times are independent. The events are neither disjoint nor exclusive.

When rolling a fair die two times, one can get any one of the 36 possible outcomes equally likely. Let A be the event of obtaining an even number on the first roll and let B be the event of getting a number greater than 3 on the second roll. Let’s see how the outcomes of A and B are related:

There are three even numbers on the die, i.e. A={2, 4, 6}. There are four numbers greater than 3 on the die, i.e. B={4, 5, 6}. So the intersection of A and B is the set {4, 6}, which is not empty. Thus, the events A and B are not disjoint. So option (A) is incorrect.

There is only one outcome that belongs to both A and B, i.e. the outcome of 6. Since there are 36 equally likely outcomes, the probability of the outcome 6 is 1/36. Now, if we know that the outcome of the first roll is an even number, does it affect the probability of getting a number greater than 3 on the second roll? Clearly not, since A∩B = {4, 6} and P(B|A) = P(A∩B)/P(A) = (2/36)/(18/36) = 1/9 = P(B). So the events A and B are independent. Thus, option (C) is correct. Neither option (A) nor option (C) can be correct, so we can eliminate options (D) and (E).

To learn more about probability, refer:-

https://brainly.com/question/31828911

#SPJ11

Find limit using Limit's properties. 3 (x+4)2 +ex - 9 lim X-0 X

Answers

The limit of the function (x+4)^2 + e^x - 9 as x approaches 0 is equal to 8.

To find the limit of a function as x approaches a specific value, we can use various limit properties. In this case, we are trying to find the limit of the function (x+4)^2 + e^x - 9 as x approaches 0.

Using limit properties, we can break down the function and evaluate each term separately.

The first term, (x+4)^2, represents a polynomial function. When x approaches 0, the term simplifies to (0+4)^2 = 4^2 = 16.

The second term, e^x, represents the exponential function. As x approaches 0, e^x approaches 1, since e^0 = 1.

The third term, -9, is a constant term and does not depend on x. Thus, the limit of -9 as x approaches 0 is -9.

By applying the limit properties, we can combine these individual limits to find the overall limit of the function. In this case, the limit of the given function as x approaches 0 is the sum of the limits of each term: 16 + 1 - 9 = 8.

Learn more about polynomial here:

https://brainly.com/question/11536910

#SPJ11

An oil company is bidding for the rights to drill a well in field A and a well in field B. The probability it will drill a well in field A is 40%. If it does, the probability the well will be successful is 45%. The probability it will drill a well in field B is 30%. If it does, the probability the well will be successful is 55%. Calculate each of the following probabilities: a) probability of a successful well in field A, b) probability of a successful well in field B. c) probability of both a successful well in field A and a successful well in field B. d) probability of at least one successful well in the two fields together,

Answers

a) The probability of a successful well in field A is 18%.
b) The probability of a successful well in field B is 16.5%.
c) The probability of both a successful well in field A and a successful well in field B is 7.2%.
d) The probability of at least one successful well in the two fields together is 26.7%.

To calculate the probabilities, we use the given information and apply the rules of conditional probability and probability addition.
a) The probability of a successful well in field A is calculated by multiplying the probability of drilling a well in field A (40%) with the probability of success given that a well is drilled in field A (45%). Therefore, the probability of a successful well in field A is 0.4 * 0.45 = 0.18 or 18%.
b) Similarly, the probability of a successful well in field B is calculated by multiplying the probability of drilling a well in field B (30%) with the probability of success given that a well is drilled in field B (55%). Hence, the probability of a successful well in field B is 0.3 * 0.55 = 0.165 or 16.5%.
c) To find the probability of both a successful well in field A and a successful well in field B, we multiply the probabilities of success in each field. Therefore, the probability is 0.18 * 0.165 = 0.0297 or 2.97%.
d) The probability of at least one successful well in the two fields together can be calculated by adding the probabilities of a successful well in field A and a successful well in field B, and subtracting the probability of both wells being unsuccessful (complement). Thus, the probability is 0.18 + 0.165 - 0.0297 = 0.315 or 31.5%.
By applying the principles of probability, we can determine the probabilities for each scenario based on the given information.

Learn more about probability here
https://brainly.com/question/31828911



#SPJ11

Consider a zero-sum 2-player normal form game given by the matrix -3 5 3 10 A = 7 8 4 5 4 -1 2 3 for player Alice and the matrix B= -A for the player Bob. In the setting of pure strategies: (a) State explicitly the security level function for Alice and the security level function for Bob. (b) Determine a saddle point of the zero-sum game stated above. (c) Show that this saddle point (from (2)) is a Nash equilibrium.

Answers

The security level function is the minimum expected payoff that a player would receive given a certain mixed strategy and the assumption that the other player would select his or her worst response to this strategy. In a zero-sum game, the security level function of one player is equal to the negation of the security level function of the other player. In this game, player Alice has matrix A while player Bob has matrix B which is the negative of matrix A.

In order to determine the security level function for Alice and Bob, we need to find the maximin and minimax values of their respective matrices. Here, Alice's maximin value is 3 and her minimax value is 1. On the other hand, Bob's maximin value is -3 and his minimax value is -1.

Therefore, the security level function of Alice is given by

s_A(p_B) = max(x_1 + 5x_2, 3x_1 + 10x_2)

where x_1 and x_2 are the probabilities that Bob assigns to his two pure strategies.

Similarly, the security level function of Bob is given by

s_B(p_A) = min(-x_1 - 7x_2, -x_1 - 8x_2, -4x_1 + x_2, -2x_1 - 3x_2).

A saddle point in a zero-sum game is a cell in the matrix that is both a minimum for its row and a maximum for its column. In this game, the cell (2,1) has the value 3 which is both the maximum for row 2 and the minimum for column 1. Therefore, the strategy (2,1) is a saddle point of the game. If Alice plays strategy 2 with probability 1 and Bob plays strategy 1 with probability 1, then the expected payoff for Alice is 3 and the expected payoff for Bob is -3.

Therefore, the value of the game is 3 and this is achieved at the saddle point (2,1). To show that this saddle point is a Nash equilibrium, we need to show that neither player has an incentive to deviate from this strategy. If Alice deviates from strategy 2, then she will play either strategy 1 or strategy 3. If she plays strategy 1, then Bob can play strategy 2 with probability 1 and his expected payoff will be 5 which is greater than -3. If she plays strategy 3, then Bob can play strategy 1 with probability 1 and his expected payoff will be 4 which is also greater than -3. Therefore, Alice has no incentive to deviate from strategy 2. Similarly, if Bob deviates from strategy 1, then he will play either strategy 2, strategy 3, or strategy 4. If he plays strategy 2, then Alice can play strategy 1 with probability 1 and her expected payoff will be 5 which is greater than 3. If he plays strategy 3, then Alice can play strategy 2 with probability 1 and her expected payoff will be 10 which is also greater than 3. If he plays strategy 4, then Alice can play strategy 2 with probability 1 and her expected payoff will be 10 which is greater than 3. Therefore, Bob has no incentive to deviate from strategy 1. Therefore, the saddle point (2,1) is a Nash equilibrium.

In summary, we have determined the security level function for Alice and Bob in a zero-sum game given by the matrix -3 5 3 10 A = 7 8 4 5 4 -1 2 3 for player Alice and the matrix B= -A for the player Bob. We have also determined a saddle point of the zero-sum game and showed that this saddle point is a Nash equilibrium.

To know more about Nash equilibrium.

https://brainly.com/question/28903257

#SPJ11

Other Questions
Jolly Cleaners offets residential and commerclal cleaning services, Clients pay a fixed monthly fee for the service, but can cancel the service at the end of any month. In addition to the employees who do the actual cleaning. the firm includes two managers who handie the administrative tasks (human resources, accounting. and so on) and one dispatcher, who assigns the cleaning employees to jobs on a dally basis. On average, residential clients pay $320 per month for cleaning services and the commercial clients pay $1,800 per month. A typical residential client requires 10 hours a month for cleaning and a typical commercial client requires 50 hours a month. in March, Jolly Cleaners had 40 commercial clients and 190 residential clients. Cleaners are paid $15 per hour and are only paid for the hours actually worked. Supplies and other variable costs are estimated to cost. 55 per hour of cleaning. Other monthly costs (all fixed) are $56,000.5G8A, including managerial and dispatcher salaries, and $3.600 in other expenses. For July. Jolly Cleaners has budgeted profit of $4.000 based on 60 commercial clients. Required: How many residential clients are budgeted for July? Low unit production cost is crucial for generating a positive gross margin. Which strategy below is NOT helpful to lower unit cost?Group of answer choicesA) Utilizing production capacityB) Higher product varietyC) Shorter setup timeD) Larger batch sizeYou are a production manager. You intend to convert the planned orders to production orders through CO41. However, the command cannot go through and there is a red cross on the planned order. Which one could be the reason?Group of answer choicesA) You did not run MRP.B) Raw materials have not been delivered.C) You run out of cash.D) There are too many scheduled production orders.Based on the Hershey case, which one is not a system that Hershey planned to implement?Group of answer choicesA) ManugisticsB) SiebelC) SAPD) Microsoft Dynamics Sort the following terms into the appropriate category. Independent Variable Input Output Explanatory Variable Response Variable Vertical Axis Horizontal Axis y I Dependent Variable what is the expression in factored form 4x^2+11x+6 the dramatic structure that features a restricted number of scenes is cannabis has been viewed as a medical treatment for __________. Ethical Practice in Real Estateb. Ethical practice standards for privacy, confidentiality and security of customer informationIn general terms describe what ethical considerations you would take account of when considering the issues of privacy, confidentiality and security of customer information and then discuss what the legislation requires you to do when handing customer information (make special reference to the Privacy Principles) Properties of Loga Express as a single logarithm and, if possible, simplify. 3\2 In 4x-In 2y^20 5\2 In 4x8-In 2y20 = [ (Simplify your answer.) 1. Water Works Plumbing Company is a small owner-managed plumbing services company that serves the greater Miami metropolitan area. Identify each of the following costs as either a variable, a fixed, or a quasi-fixed cost and give a detailed explanation. a) Gasoline expense for the service van. b) Cost of the owner's time to run the plumbing business. c) Cost of a complete set of tools needed to be a plumber. d) Labor expense for an assistant plumber who is hired on an hourly basis and works with the owner-manager of the firm when the owner needs a helper. HSave Assignment Submitted Back e) Monthly lease payment for a drain-line auger, which contractually binds WW Plumbing to pay $75 per month for the next 12 months, regardless of how much or how little the company uses the leased piece of plumbing equipment. Subleasing is prohibited and there will be no refund if the machine is returned before the 12 month period expires. f) Expense for plumbing service consumables: plumbers' putty, Teflon tape, pipe lubricant, sandpaper, PVC glue, butane for torch, etc. When using a periodic inventory system, Cost of Goods Sold and the Inventory accounts are updated:a. when cash is received.b. when revenue is earned.c. when a sale is made.d. when a count is taken. Let A = {2, 4, 6} and B = {1, 3, 4, 7, 9}. A relation f is defined from A to B by afb if 5 divides ab + 1. Is f a one-to-one function? funoti Show that Assume that x and y are both differentiable functions of t and find the required values of dy/dt and dx/dt. xy = 2 (a) Find dy/dt, given x 2 and dx/dt = 11. dy/dt = (b) Find dx/dt, given x-1 and dy/dt = -9. dx/dt = Need Help? Read It 2. [-/3 Points] DETAILS LARCALCET7 3.7.009. A point is moving along the graph of the given function at the rate dx/dt. Find dy/dt for the given values of x. ytan x; - dx dt - 3 feet per second (a) x dy W ft/sec dt (b) dy dt (c) x-0 dy dt Need Help? Read It 3. [-/1 Points] DETAILS LARCALCET7 3.7.011. The radius r of a circle is increasing at a rate of 6 centimeters per minute. Find the rate of change of the area when r-39 centimeters cm2/min. X- - 71 3 H4 ft/sec ft/sec Why does the author choose to have Jonas's father speak to the twin the same way he speaks to Gabe? A. To show that Jonas's father thinks the twin could be good friends with Gabe.B. To show that Jonas's father is childish and speaks in that tone all day at work.C. To show that Jonas's father thinks what he's doing is completely normal and okay.D. To show that Jonas's father wants to take the twin home and care for him like he has for Gabe. One of Ed's favorite bands is playing in Philadelphia. Ed purchases a ticket ($50.00) and takes a day off work to get ready for the concert (Ed earn $75.00). While standing on line to get into the venue, someone offers Ed $160 for his ticket, but he turns them down. From this, we can infer that the benefit Ed gets from attending the concert is at least dollars (please record your answer without a dollar sign). 10 points Now recall the method of integrating factors: suppose we have a first-order linear differential equation dy + a(t)y = f(t). What we gonna do is to mul- tiply the equation with a so called integrating factor . Now the equation becomes (+a(t)y) = f(t). Look at left hand side, we want it to be the dt = a(t)(explain derivative of y, by the product rule. Which means that d why?). Now use your knowledge on the first-order linear homogeneous equa- tion (y' + a(t)y = 0) to solve for . Find the general solutions to y' = 16 y(explicitly). Discuss different inter- vals of existence in terms of different initial values y(0) = y the first africans in colonial america were indentured servants whose children were ________. When discussing and evaluating professional ethics, it is essential to understand the purpose, terminology, and repercussions of professional misconduct. The American Institute of Certified Public Accountants (AICPA) code of professional conduct is the gold standard for defining professional conduct in accounting; it is therefore important for business professionals to be familiar with. In this discussion, you will explore one principle in depth and discuss it and others with your peers.First, select one of the following principles of professional conduct to examine in the AICPA Code of Professional Conduct document:ResponsibilitiesPublic interestIntegrityObjectivity and independenceDue careScope and nature of servicesThen, for your initial post, reflect on what appropriate practice of your selected principle would look like in the field, and also on some potential examples of violations of the principle. Use the following questions to help guide your reflections:How would you define and describe your selected principle in your own words?What value does the principle bring to practitioners, businesses, and clients?What is an example of a difficult situation that a practitioner may face related to your selected principle, and what would an ethical response to the situation be? Why might a practitioner be tempted to, or accidentally, not take an ethical course of action? Hunt Company purchased factory equipment with an invoice price of $60,000. Other costs incurred were freight costs, $1,100; installation, $2,200; labor in testing equipment, $700; fire insurance policy covering equipment, $1,400. The equipment is estimated to have a $5,000 salvage value at the end of its 10 year useful life. Instructions: a) Compute the acquisition cost of the equipment _____ b) If the double-declining balance method of depreciation was used, the percentage applied to a declining book value would be ____ Dolvin Industries produces electronic equipment for use in small aircraft. Last years sales totaled $675,000, variable costs $70,000, fixed costs $20,000 and depreciation $115,000. Over the upcoming year, sales and variable costs are expected to rise 20 percent while fixed costs and depreciation are expected to be constant. Some time ago, Dolvin had purchased land at a cost of $260,000 and now wants to utilize the land for building another factory that will produce small aircraft navigational equipment. If it decides to go ahead and construct the new factory, it will carry an upfront cost of $600,000 and take two years to construct. The machinery and installation necessary to begin production would cost $790,000 which would be paid after the factory is constructed. Both the plant and equipment would be depreciated on a straight-line basis over the 4-year life of production, for which at the end of that time, the property and plant could be sold for $600,000 and the machinery scrapped for $150,000. Estimated sales from production would be $850,000 per year with $90,000 of that amount being variable cost. The annual fixed cost would be $25,000. The project will require $10,000 of net working capital which is recoverable at the end of the project. The firm's discount rate for a project of this risk is 12 percent. Another option available to Dolvin is that the land could be sold to a buyer that is willing to pay cash upfront of $500,000. The company's tax rate is 34 percent.1. If Dolvin decides to build the new factory, answer the following:a. What is the proper cash flow amount to use as the initial investment? Show your computations.b. What are the proper cash flow amounts that will occur over each of the 4 years of production? Show your computations.c. What is the net present value? Show your computations.2. Would it be rational instead for Dolvin to sell the land? Explain. The following data pertains to CEC Corp. + CEC Corp. Total Assets Interest-Bearing Debt (market value) Average borrowing rate for debt Common Equity: Book Value Market Value Marginal Income Tax Rate Market Beta $23,610 $11,070 12% $6,150 $25,830 25% 2.5 1. Using the information from the table, and assuming that the risk-free rate is 5% and the market risk premium is 4%, calculate CEC's cost of equity capital from using the CAPM and cost of debt capital: 2. Using the information from the table, calculate CEC's weighted-average cost of capital: